Topologia e conjuntos em exercícios

Mantido pelo grupo "Topologia do Interior"

Ferramentas do usuário

Ferramentas do site


lista:pontofixo

Teoremas de ponto fixo

Provavelmente você vai querer saber os resultados da lista de sequências de Cauchy e completude.

Seja $(X, \tau)$ um espaço topológico. Dizemos que $x \in X$ é um ponto/fixo; ponto fixo para $f: X \rightarrow X$ se $f(x) = x$.

Sejam $(X, d)$ e $(Y, d')$ espaços métricos. Dizemos que $f: X \rightarrow Y$ é uma contração se existe $k \in [0, 1[$ tal que, para todo $x, y$ distintos, $d'(f(x), f(y)) \leq k d(x, y)$.

1 Mostre que toda contração é uma função contínua.

2 Seja $(X, d)$ espaço métrico e seja $f: X \rightarrow X$ uma contração. Mostre que, se $f$ admite um ponto fixo, ele é único.

3 Seja $(X, d)$ espaço métrico completo e seja $f: X \rightarrow X$ uma contração. Seja $x_0 \in X$. Para cada $n \in \mathbb N_{>0}$, defina $x_{n + 1} = f(x_n)$.

3.1 Seja $n \in \mathbb N$. Mostre que $d(x_n, x_{n + 1}) \leq k^n d(x_0, x_1)$.

3.2 Sejam $n, p \in \mathbb N$. Mostre que $d(x_n, x_{n + p}) \leq \frac{k^n}{1 - k} d(x_0, x_1)$.

3.3 Conclua que $(x_n)_{n \in \mathbb N}$ é uma sequência de Cauchy e, portanto, convergente para algum $x \in X$. Note que tal $x$ é um ponto fixo.

3.4 Note que você mostrou o Teorema/ponto fixo de Banach; Teorema do ponto fixo de Banach: Dado $(X, d)$ métrico completo e $f: X \rightarrow X$ contração, então $f$ admite um único ponto fixo $x$. Além disso, dado $x_0 \in X$, se definimos $x_{n + 1} = f(x_n)$ para todo $n \in \mathbb N$, temos que $x_n \rightarrow x$.

Vamos ver que a hipótese $k < 1$ para a contração é essencial para o teorema do ponto fixo de Banach. Também veremos que nem tudo está perdido se retirarmos isso, mas daí você provavelmente vai querer saber os resultados da lista de compactos.

4 Considere $X = [1, +\infty[$ com a métrica usual. Considere $f: X \rightarrow X$ dada por $f(x) = x + \frac{1}{x}$.

4.1 Note que $X$ é completo.

4.2 Note que $f$ não admite ponto fixo.

4.3 Note que $f$ é estritamente crescente.

4.4 Mostre que, dado $x, y \in X$ distintos, temos que $d(f(x), f(y)) < d(x, y)$.

5 Seja $(X, d)$ compacto. Seja $f: X \rightarrow X$ tal que $d(f(x), f(y)) < d(x, y)$ se $x \neq y$.

5.1 Note que $f$ é contínua.

5.2 Note que, se $f$ tem ponto fixo, então ele é único.

5.3 Note que $g: X \rightarrow \mathbb R$ dada por $g(x) = d(x, f(x))$ é contínua.

5.4 Mostre que $f$ tem um ponto fixo.

lista/pontofixo.txt · Última modificação: 2020/11/06 16:05 (edição externa)